If 7 > 2, then explain why -7 < -2.
A. The above statement is false.
B. Because the sign changed.
C. Because -7 is more negative and further to the left on the numberline than
-2.
D. Because 7 is an odd number and 2 is even, which flips when they become
negative, where -7 becomes even and -2 becomes odd.

Answers

Answer 1

Answer:

C

Step-by-step explanation:

Hope it can help you lovelots

Answer 2

If 7 > 2, then explain why -7 < -2.

Answer =》C. Because -7 is more negative and further to the left on the numberline than

-2.

______

RainbowSalt2222 ☔

If 7 &gt; 2, Then Explain Why -7 &lt; -2.A. The Above Statement Is False.B. Because The Sign Changed.C.

Related Questions

How much money will be in an account with an initial investment of $13, 675 at 4.3% interest over 5
years if the interest is compounded annually?

Answers

The end balance in the account will be$16,948.6 and the total compound interest is $3,273.6. Does that help?

What is 1/6 to the power of 3

Answers

Answer:

[tex]\frac{1}{216}[/tex]

Step-by-step explanation:

Exponents on fractions work like this:

[tex](\frac{x}{y} )^a=\frac{x^a}{y^a}[/tex]

Apply that to your problem:

[tex](\frac{1}{6} )^3\\\\\frac{1^3}{6^3} \\\\\frac{1}{216}[/tex]

Answer:

1/216

Step-by-step explanation:

1/6 x 1/6 is 1/36

1/36 x 1/6 is 1/216

1. f(x) = x2
• vertex :
• opening of the graph :
• vertex is a ______ point
• equation of axis of symmetry (x=h) :
• range (y≥k) :​

Answers

Answer:

Step-by-step explanation:

f(x) = x²

1. Vertex ( 0 ,0)

2) a > 0 , opening upward

3) minimum

4) Axis of symmetry : x = 0

5) Range:  y ≥ 0  

Sam and Andre shared a number of pennies in the ratio 8:3. They received a total of 671 pennies. How many more pennies did Sam receive than Andre?

Answers

Answer:

Sam had 488

Andre had 183

488:183

please mark brainllest

The marked price of a stove at Courts was $1,200. A lady bought the stove by making a deposit of $400, and paying $80 per month for one year and a half. How much would she have saved by paying cash?

Answers

she would have saved $650

What is the range of this relation?
(-5,0)
(8,9)
(-10, 2)
(-1, -3)

Answers

Answer:

{0,9,2,-3}

Step-by-step explanation:

the range is all the ys.

these are all in the format (x, y)

Hope this helps.

PS-If it's a multiple choice question, the numbers might not be in the same order. just pick the one with those numbers.


Choose all of the proportional relationships that have a greater unit rate than the unit rate of this graph.
10
9
8
7
6
5
4
3
2
1
5
7
B
00
9 TD
X
y
2
5
10
15
low
y
3
6
9
6
12
18
6
y = 2x
y = 0.25%
y
2
4
y
3
6
9
8
16
24
8
12

Answers

The proportional relationships that have a unit rate that is greater than that of the one of the graph are: the first and third tables, and y = 2x.

How to Determine the Unit Rate of a Proportional Relationship?

The unit rate can be calculated as, m = y/x. In an equation that represents a proportional relationship, y = mx, the value of m is the unit rate.

Using a point on the graph, (5, 2):

Unit rate of the graph = y/x = 2/5

Unit rate = 0.4.

Unit rate for the first table = 3/5 = 0.6 [greater than 0.4]

Unit rate for the second table = 2/6 = 0.3 [less than 0.4]

Unit rate for y = 2x, is 2 [greater than 0.4].

Unit rate for y = 0.25x, is 0.25 [less than 0.4].

Unit rate for the third table = 2/4 = 0.5 [greater than 0.4]

Unit rate for the second table = 3/8 = 0.375 [less than 0.4]

Those that have a unit rate that is greater than the one of the graph are: the first and third tables, and y = 2x.

Learn more about unit rate on:

https://brainly.com/question/4895463

#SPJ1

9. Allen and his two brothers decide to purchase a video game system together. They determine
that each person will need to pay $32.30. How much is the cost of the video game system?
I need a variable, equation, and the solution. Pls help!!!

Answers

Step-by-step explanation:

that is so simple, it is already difficult to make it formal, because there are only so few things to deal with. you really need help with this ?

you are not familiar with the situation "hey, this costs $10, we are 5 people, if we buy and then use it together, each one has to pay $2" ?

and this is just the other way around : if 5 people paid $2, how much did they pay together ? $10.

let's make it super formal :

price p = Allen contribution a + brother1 contribution b + brother2 contribution c

p = a + b + c

a = b = c = $32.30

p = $32.30 + $32.30 + $32.30 = 3×$32.30 = $96.90

Each big square below represents one whole.

Answers

Answer:

103%

Step-by-step explanation:

each little box is 1%

Using the formula find the slope of a line with the points (10,20) and (30,50).

Answers

Answer:

m = 2/3

Step-by-step explanation:

Slope is defined as the ratio rise/run.

As we move from (10, 20) to (30, 50), the run is 30 - 10, or 20, and the rise is 50 - 20, or 30.  Thus, the slope of the line is m = 30/20, or 2/3.

can y⁵ x y be simplified??​

Answers

Answer:

Yes,

[tex]y^{6} x[/tex]

Step-by-step explanation:

[tex]y^{5} xy = y^{5} *y*x = y^{6} x[/tex]

Find the value of x and y

Answers

Answer:

x=4

y=5

Step-by-step explanation:

because this lines are all parallel, so 17x-y equals to 180°-117°= 63, then you get 17x-y = 63

also, equally you can get the below one

6x+7y = (180°-121°)

6x+7y = 59.

put them together

17x-y=63 (1)

6x+7y =59 (2)

you can transform the (1) to y= -(63-17x)

then you get the second one

6x+7(-(63-17x))=59

then you get x

finally you get the result of x into any of the equation

you get y ultimately.

QUICK!!!! I NEED AN ANSWER!! Emily says that the negative sign means opposite of, so -(-45) means the opposite of negative 45, which is 45. Is Emily correct?

Answers

Answer:

Yes, Emily is correct.

Step-by-step explanation:

If you have a - sign in front of a number in parenthesis that is negative, then what you are doing is taking the opposite of that negative number. Remember that the opposite of a negative number is a positive number. In this case, we want the opposite of negative five. What is that? It is +45, or 45. So yes, Emily is correct.

Half the sum of a number and 1.75 is −5.1. Enter your answer as a simplified mixed number

Answers

The unknown number expressed as a mixed fraction is 11 19/20

Let the unknown number be "x"

The sum of the number and 1.75 is expressed as x + 1.75

Half the sum of a number and 1.75 is 1/2(x+1.75)

Equating the result to -5.1 wil give:

1/2(x+1.75) = -5.1

Solve the resulting equation

x + 1.75 = 2(-5.1)

x + 1.75  = -10.2

x = -10.2 - 1.75

x = 11.95

x = 1195/100

x = 11 95/100

Hence the unknown number expressed as a mixed fraction is 11 19/20

Learn more here: https://brainly.com/question/14583740

Answer

-11 19/20

I took the test and got it right!

2 + 2 = 4 - 1 =






















thats 3 quick maffs

Answers

Answer:

3

Step-by-step explanation:

-3(u+4)-2u=2(u+1)
a) no solution
b) u=
c) All real numbers are solution

Answers

Answer: u = -2

Step-by-step explanation:

-3(u+4)-2u=2(u+1)

-3u-12-2u=2u+2

-5u-12=2u+2

-7u = 14

u = -2

I NEED HELP please, take 50 points if you answer It and a 50/50 at a brainiest.

"The hourly wage increase each employee receives each year depends on their number of years of service. Every three years of service means an increase of $0.50 per hour. So, employees that have been with the company for less than three years can expect to receive an increase of $0.50 per hour. Employees that have been with the company for at least three years, but less than six years can expect an increase of $1.00. Employees that have been with the company for at six years, but less than nine years, receive an increase of $1.50 per hour. And, employees of at least nine years, but less than twelve years receive an increase of $2.00. Write a function to represent this scenario.

Which graph represents this wage increase for x < 12?"

-normally I'd just double-check this but. . . you get it.

Answers

Red dots are starting co-ordinates and whites are ending co-ordinates .

So the graph must have red dots at the start of line and not mandatory for at the end of line.

x<12

It should not be at end

Check option B

x ends at first point .

And also the interval is 2

so Option B is correct

Slope help please it is supposed to be easy

Answers

Look at the coordinates, y=4 in that coodinate meaning it is the y=intercept.

The slope u already know,

The formula for equation --> Y=mx+b

Answer: Y=3/4+4

A mixture of paint calls for 1/3 cups of red paint and 1/2 cup of yellow paint. How many cups of red paint would be needed for every 1 cup of yellow paint?

Answers

Step-by-step explanation:

1 cup of yellow paint = 2×1/2 cup.

and that requires then also 2×1/3 Co red paint.

so, 1 cup of yellow paint mixes with 2/3 cups of red paint.

2 and 3 have a smallest common multiple (smallest number that is divisible by both numbers without remainder) of 6.

so, we need to get 6 units of each for a whole number ratio :

6×1/2 = 3 cups of yellow paint mix with 6×1/3 = 2 cups of red paint.

How much water can the large water hold?

Answers

Answer:

700 ml

Step-by-step explanation:

If the 15cm bottle holds 500ml of water, you can create the ratio of 15:500, which simplifies to 3:100.

Since the second bottle is 21cm, you find 21:x.

Because you already have 3:100 as given from the first bottle, if you multiply both sides by 7 you get 21:700.

This means the second bottle holds 700ml.

Answer:

700 mi of water

Step-by-step explanation:

We can divide 21 by 15 to how many times greater they are to each other

21÷15=1.4

Since the 15 cm bottle of water can hold 500 mi we multiply 500 • 1.4 = 700mi

Brainliest please

I’m really bad at math please help me

Answers

x=20

Hope this helps!!

The best way to accurately solve real-life problems involving decimals is through/by?

Answers

Answer:

Rounding maybe

Step-by-step explanation:

a) Solve the system by substitution.
y = 5x – 3
-X – 5y = -11

Answers

Answer:

Step-by-step explanation:

the answer 4x-5y +2 = 0. 5y =-4x-2. y = -4x/5 -2/5 slope = -4/5

Answer:

{y,x} = {2,1}

Step-by-step explanation:

// Solve equation [1] for the variable  y  

 

 [1]    y = 5x - 3

// Plug this in for variable  y  in equation [2]

  [2]    -5•(5x-3) - x = -11

  [2]     - 26x = -26

// Solve equation [2] for the variable  x  

  [2]    26x = 26  

  [2]    x = 1  

// By now we know this much :

   y = 5x-3

   x = 1

// Use the  x  value to solve for  y  

   y = 5(1)-3 = 2

Complete the table using the equation y = 8x + 8 ( helppp plsss)

Answers

Answer:

0, 8, 16, 24, 32

Step-by-step explanation:

plug in the values given for x into the 8x+8 equation

Example:

8(-1)+8

8(0)+8

Answer:

(-1, 0), (0, 8), (1, 16), (2, 24), (3, 32)

Step-by-step explanation:

To find answers to problems like these, take the x-value, which in this case is -1, and replace it with the variable,

y = 8*-1 + 8. Then do the rest, y = -8 + 8, y = 0. You already have the x, -1, and now you got y, 0. Then pit in ordered pairs, x first, then y, (-1, 0)

Raffle tickets are being sold for a fundraiser. The function a(n) relates the
amount of money raised to the number of tickets sold, n.
It takes as input the number of tickets sold and returns as output the amount
of money raised.
a(n) = 3n - 30
Which equation represents the inverse function n(a), which takes the money
raised as input and returns the number of tickets sold as output?

Answers

The money raised depends only on the number of tickets sold, the inverse equation is found by making the tickets sold the subject.

[tex] \underline {n =\frac{a(n) }{3} + 10}[/tex]

How can the inverse function be found?

The given parameters are;

a(n) = The amount of money raised

n = The number of tickets sold

a(n) = 3•n - 30

Solution;

The equation that takes the amount of money raised and returns the number of tickets sold is found as follows;

a(n) = 3•n - 30

a(n) + 30 = 3•n

Therefore;

[tex]n = \frac{a(n) + 30}{3} = \frac{a(n) }{3} + 10[/tex]

The number of tickets sold,

[tex] \underline {n =\frac{a(n) }{3} + 10}[/tex]

Learn more about equations here: https://brainly.com/question/24714911

#SPJ1

i need some help with simple algebra! using pemdas :p

Answers

Answer:

the answer is 69

Step-by-step explanation:

the step by step explanation is that 2+2+4

pemdas

perenthasy

exponents

multiplication

division

adition

subtraction

Consider the number 9,953. Is this number divisible by 2, 3, 4, 5, 9, or 10? Provide justification for each number without using a calculator. Hint: If a number is not divisible by 2, then what other numbers is it also not divisible by?

Answers

Answer:

Divisible by none

Step-by-step explanation:

9,953 is odd

9+9+5+3 = 2 + 6 = 8 (not divisible)

53/4 is not a whole number

9,953 doesn't end with 5 or 0.

9 + 9 + 5 + 3 = 26 (not divisible by 9)

9,953 has no 0 at end.

You are welcome!

Kayden Kohl

8th Grade Student

The number 9,953 is divisible by neither of the digits 2, 3, 4, 5, 9, 10.

What is the importance of knowing divisibility rules?

The importance of knowing divisibility rules is that we can determine whether a number is divisible by certain numbers generally from 1 to 20 or not without actually dividing.

Given, 9,953. Is this number divisible by 2, 3, 4, 5, 9, 10 or not.

A number is divisible by 2 if it's unit digit is divisible by 2.

A number is divisible by 3 if the sum of the digits is divisible by 3.

A number is divisible by 4 if the last two digits is divisible by 4.

A number is divisible by 5 if its unit digit is 5 or 0.

A number is divisible by 9 if the sum of the digits is divisible by 9.

A number is divisible by 10 if its unit digit is 0.

learn more about divisibility rules here :

https://brainly.com/question/10703980

#SPJ2

Please help me please help meeee please

Answers

Answer:

The answer is D

Step-by-step explanation:

Solve by substitution. x=y-9,5x+2y=11

Answers

We have two equations :

x=y-9

5x+2y = 11

We just have to plug the value of x in the second equation to find y :

5(y-9) + 2y = 11

5y - 45 + 2y = 11

7y - 45 = 11

7y = 11 + 45

7y = 56

y = 56/7 = 8

Finally, since x = y -9

x = 8-9 = -1

Good Luck

Answer:

X = -1 Y = 8

try downloading mathaway it might help with these problem types

Help me with this hw plz

Answers

Answer:

here

Step-by-step explanation:

А. parallelogram .............

Answer: A
Step-by-step explanation:
Other Questions
Can someone help me with this question it's my last question I will give you a brain list just help me I will really appreciate it thank you :) Help help help help help please math Im stuck Which monarch was the target of the Gunpowder Plot? A. Elizabeth I B. James I C. Charles I D. Charles II. What do you mean by Litmus : A natural dye. The function of the sputnik device was which of the following? A: Create a nuclear reactor in spaceB: Create a missile - launching deviceC: Create a satellite that gave a signal Somebody help ??????? I am good at reading not math I need help please 7. Proving the Perpendicular Bisector Theorem Using ReflectionsGiven: Point P is on the perpendicular bisector m of AB.Prove: PA PB outline 2 reasons for completing assigned tasks daily Which of the following is released during cellular respiration? Carbon dioxide Glucose Mitochondria Oxygen A flight from Barrow to Murmansk would closely follow which lines of longitude?A.120'W and 60'EB.150'W and 30'EC.180'W and 0'PLZZZ HELP Need the help nowwwwwee please any help will do 50 points question in image help WHAT WAS THE WAR IN 1775 Help help help help help help help help help WHAT IS THE EQUATION IN SLOPE INTERCEPT FORM FOT THE LINE THAT PASSES THROUGH THE POINTS (2,-2) AND (-3,-17) If f(x)=2(x2+5), then f(-3)= PLEASE HELP WILL MARK BRAINLIEST This passage is a short poem written by a yeomanette. What does it suggest about women and the us navy during the war? check all that apply. Women in the navy could explore the world. Women were discouraged from signing up. Women were stationed only in newport. Yeomanettes were held in high esteem. Yeomanettes made few contributions. A spoonful of ice cream contains 0.006 mg of iron. How much ironwould Mr. Lin consume if he ate 100 spoonfuls?